Você está na página 1de 6

Assignment I

DISTRIBUTION
November 30, 2016

Prepared By

Getachew Fetene(GSR/1411/08)
Miliyon Tilahun(GSR/1401/08)
Fatuma Zeynu(GSR/1398/08)
Adem Aman(GSR/1397/08)
Eshetu Dadi(GSR/1524/08)

X
Submitted to

Tamirat Temesgen (PhD)

Problems from worksheet


1. (Problem 1) Show that following expressions define a distribution.
hf , i =

m
X

n (0),

n=1

Solution. i) Linearity: Let 1 , 2 D and , R.


hf , 1 + 2 i =

m
X

(1 + 2 )n (0)

n=1

m
X

1n (0) + 2n (0)

n=1

m
X

1n (0) +

n=1

m
X

2n (0)

n=1

= hf , 1 i + hf , 2 i
ii) Continuity: Let k in D (). Then there is exists E compact in such that suppk E for
all k = 1, 2, . . .. Moreover
D k D
Now,


m
m
X
X

kn (0)
n (0)
|hf , k i hf , i| =
n=1

n=1


m 

X
=
kn (0) n (0) 0 as k
n=1

This is because uniform continuity implies pointwise continuity. Thus,
hf , k i = hf , i
Therefore,
hf , i =

m
X

n (0),

n=1

is indeed a distribution.

2. (Problem 3) Prove that

d
dx

ln x = P 1x .

Solution. Let the function f (x) = ln |x|, x R\{0}. This function is locally integrable, so f L1loc (R). To
the considered function we assign the linear functional defined by the formula
Z
hln |x|, i =
(x) ln |x|dx, D (R)
R

We shall show that the functional Tf = ln |x| is a distribution on D 0 (R). We have


"Z

Z
hln |x|, i =

(x) ln |x|dx = lim

+0

Z
(x) ln |x|dx +

#
(x) ln |x|dx = lim

+0

[(x) (x)] ln xdx

Considering supp() [a, a], a > 0, then we can write


Z
|(x) (x)|| ln x|dx
hln |x|, i lim
+0

Because

lim [(x) (x)] ln x = 2 lim [ 0 (x )x ln x] = 2 0 (0) lim x ln x = 0

x+0

x+0

we have

x+0

hln |x|, i 2 sup |x ln x| sup | 0 (x)|


x[0,a]

hence

hln |x|, i c sup | 0 (x)|,

x[0,a]

where c = 2 sup |x ln x|.

x[0,a]

x[0,a]

Consequently, the functional Tf is continuous and the linearity follows from the linearity of integrals.
Hence,Tf = ln |x| is a distribution on D 0 (R).
Now, we have
0

h(ln |x|) , i = h(ln |x|), i = 0 (x) ln |x|dx


R
Z
= lim
[ 0 (x) 0 (x)] ln xdx
+0
Z
(x) (x)
= lim [() ()] ln + lim
dx
0
+0
x
Because lim0 [() ()] ln = 0 and
Z
Z
(x) (x)
(x)
1
lim
dx = hp.v. , i = p.v.
dx
0
x
x
R x
we can write

1
1
h(ln |x|)0 , i = hp.v. , i = hP , i
x
x

Therefore,
(ln |x|)0 = P

1
.
x

3. (Problem 4) Find the nth derivative of f (x) = |x|.


Solution. Let D (R), supp [a, a] for some a > 0.
f (x) = |x|
h|x|0 , i = h|x|, 0 i
Z
=
|x| 0 (x)dx
ZRa
=
|x| 0 (x)dx
Z

a
0

x (x)dx

x 0 (x)dx

a Z a
0 Z 0
(x)dx x(x) 0 +
(x)dx
= x(x) a
0
| {z } a
| {z }
Z

0
0

=
(x)dx +
(x)dx
0
Z aa
=
sgn(x)(x)dx
a

= hsgn(x), i
Therefore,

|x|0 = sgn(x)

Now,
h|x|, i = h(sgn(x))0 , i = hsgn(x), 0 i
Za
=
sgn(x) 0 (x)dx
Z

a
0

0 (x)dx

0 (x)dx

0
a
= (x) a (0) 0
= (0) + (0) = 2(0)
= h2, i
Hence, in a similar fashion

f (n) (x) = |x|(n) = 2n1 ,

for n 2.

4. (Problem 5) Consider the sequence of functions on R defined by


fn (x) =

n
,
(1 + n2 x2 )

n = 1, 2, . . .

Show that {fn } converges to in the sense of distribution.


Solution. We have
Z
lim hfn , i = lim

n(x)
dx = lim
n
(1
+ n2 x 2 )

n(x)
dx,
(1 + n2 x2 )

since is test function (and therefore (x) = 0 for x < [M, M]).
By substituting t = nx, we have
nM

Z
lim hfn , i = lim

nM

( nt )
dt = lim
n
(1 + t 2 )

[nM,nM] (t)( nt )

(1 + t 2 )

dt,

from which we get by Lebesgue dominated theorem,


Z
lim hfn , i =

lim

[nM,nM] (t)( nt )
(1 + t 2 )

(0)
(0)
dt =
arctan t|
= (0)
2

(1 + t )

dt =

lim hfn , i = (0) = h, i

Thus, {fn } converges to in the sense of distribution.


5. (Problem 8) Prove that for every continuous function a we have a = a(0), and, if a C 1 then a0 =
a(0)0 a(0). In particular for a(x) = x, show that x = 0, x(m) = mm1 for m N.
Solution. Let a(x) be a continuous function and (x) D .
i)
ha(x)(x), (x)i = h(x), a(x)(x)i
= a(0)(0)
= a(0)h, i
Therefore,
a(x)(x) = a(0)(x)

(1)

ii)
ha(x)0 (x), (x)i = h0 (x), a(x)(x)i = h0 (x), (a(x)(x))0 i
= h(x), a0 (x)(x) + a(x) 0 (x)i
= h(x), a0 (x)(x)i h(x), a(x) 0 (x)i
= ha0 (x)(x), (x)i ha(x)(x), 0 (x)i
= ha0 (0)(x), (x)i ha(0)(x), 0 (x)i
0

= ha (0)(x), (x)i + ha(0) (x), (x)i


= ha(0)0 (x) a0 (0)(x), (x)i
Therefore,

a(x)0 (x) = a(0)0 (x) a0 (0)(x)

iii) In particular if a(x) = x, a0 (x) = 1.


x(x) = 0 = 0
x0 (x) = 0 0 (x) 1 (x) = (x)
Induction hypothesis(I.H), suppose it holds for m = r 1 i.e.
xr1 (x) = (r 1)r2 (x)

(By 1)

We need to show that it holds for m = r.


hxr (x), (x)i = hr (x), x(x)i
= hr1 (x), (x(x))0 i
= hr1 (x), (x) + x 0 (x)i
= hr1 (x), (x)i hr1 (x), x 0 (x)i
= hr1 (x), (x)i hxr1 (x), 0 (x)i
= hr1 (x), (x)i h(r 1)r2 (x), 0 (x)i
= h

r1

= h

r1

= hr
Hence

(x), (x)i + h(r 1)


(x) (r 1)

r1

r1

r1

(x), (x)i

(x), (x)i

(x), (x)i

xr (x) = rr1 (x)

Therefore, by the principle of mathematical induction, we conclude that


xm (x) = mm1 (x),

The End

m N.

(By I.H)

Você também pode gostar